LSAT and Law School Admissions Forum

Get expert LSAT preparation and law school admissions advice from PowerScore Test Preparation.

 emilysnoddon
  • Posts: 64
  • Joined: Apr 22, 2016
|
#26298
I had a difficult time with this question. I chose answer choice C because I thought the part theauthor said that biotechnology researchers who work in academic institutions support intellectual property protection because the research funding is conditional on the patentability of their results provided support. I realize why this is wrong but I do not understand where the support in the passage is for answer choice D. Can someone please point me in the right direction?

Thank you,

Emily
User avatar
 Jonathan Evans
PowerScore Staff
  • PowerScore Staff
  • Posts: 726
  • Joined: Jun 09, 2016
|
#26354
Emily, you raise two interesting points.

First, with respect to answer choice (C), you have noted that the author does make a connection between the patentability of results and the availability of research funding. The connection between this answer choice and the information in the passage bears a brief discussion. As this is a Must Be True question (i.e. a question that asks you to prove something based on information in the passage), any time you find yourself reacting to an answer choice thinking, "Well, that seems likely" or "Possibly, since there is a connection between those two things," you need to pause and ask yourself, "Do I know this for sure?" Attractive wrong answers on Must Be True questions will often make students think, "Hey this looks good to me!" You need to approach these questions with an appropriate degree of skepticism and scrutiny. Zero in on words such as "generally" in answer choice (C), and verify whether the strength of this word is provable based on the passage. Also, look to make sure you have definite information about biotechnology researchers who oppose biotechnology patents. To riff off an old adage, when it comes to Must Be True, don't trust, and verify.

Second, with respect to answer choice (D), start by noting the contrast in the strength of language here. Instead of "generally" as in (C), we have "not the only way." This kind of qualifier indicates a less extreme, i.e. easier to prove statement (it's easier to show that at least one swan is white than to show that all swans are white). Now look for the explicit support for answer choice (D) in the passage. Start by recalling the organization of the passage. Where in the passage does the author first detail some methods which patent holders can use to enforce restrictions? The second paragraph is a natural place to begin since it is where the author expounds upon the "threat to basic research" from patents. Following the directional indicators "some researchers," then "in other instances," and finally "for example," which directly addresses the preceding "materials-transfer agreement" situation. You can notice the progression of this topic. The support for this answer is found in lines 23-34.

In lines 19-23, the author describes the method of patent holders' suing for patent infringement to assert legal control, but after that in 23, the "in other instances" indicated we are now out of lawsuit land and in a different situation. The situation detailed is that of restricting access through materials-transfer or license agreements. The passage further discusses the prohibitively high fees that patent holders could potentially charge to restrict access to (i.e. "assert legal control over the use of")their patented materials.

Based on the evidence above, there is another way besides suing for patent holders to assert legal control over the use of their patented materials.

I hope this explanation clears up some of your questions. Please reply if you need further help with this or any other question.
 Lourdiana
  • Posts: 14
  • Joined: Oct 09, 2018
|
#60169
Good Evening and sorry if this is posted twice! I thought I sent this question but not sure what happened. I wanted to know if B was a mere statement versus a support? I seemed to have chosen B and now looking back, I think its because the information in B seemed most familiar - any help would be great.

Thanks,
Lourdi
 Robert Carroll
PowerScore Staff
  • PowerScore Staff
  • Posts: 1787
  • Joined: Dec 06, 2013
|
#61573
Lourdi,

The question is asking for something that is supported by the passage. If a particular statement is made by the passage, the passage does support that. So there is not a distinction between "statement" and "support" in this case. It would be different if the question were asking whether the function of one thing was to support another in the passage. Here, you're just looking for anything the passage supports.

Lines 47-51 do not state or support what answer choice (B) says. We do not know that most patent holders believe there is a threat to their market position. We know instead that the author believes they will usually engage in a certain action only when they have that belief. How often they have that belief is unknown. Thus, answer choice (B) cannot be inferred.

Robert Carroll
User avatar
 Albertlyu
  • Posts: 98
  • Joined: Jul 18, 2020
|
#79071
hi thanks, what really confused me is what classifies as asserting legal control over the use of their materials. So selling or requiring users to be licensed is asserting legal control? can someone please help? thanks.
User avatar
 KelseyWoods
PowerScore Staff
  • PowerScore Staff
  • Posts: 1079
  • Joined: Jun 26, 2013
|
#79810
Hi Albert!

Yes, "material-transfer agreement or license agreement" refers to other ways, besides lawsuits, which patent holders can use to assert legal control over the use of their patented materials. They are controlling who has the ability to use their materials and this control is backed by laws and legal documents.

Hope this helps!

Best,
Kelsey

Get the most out of your LSAT Prep Plus subscription.

Analyze and track your performance with our Testing and Analytics Package.